Question

Suppose q=L1/3K1/6A1/18. You get to choose the L,K, and A and the cost of these inputs are w, r, and m, respectively. a-...

Suppose q=L1/3K1/6A1/18. You get to choose the L,K, and A and the cost of these inputs are w, r, and m, respectively.

a- what type of returns to scale does this production function have?

b- write down the lagrange for the cost minimization problem and find the first order conditions.

c- Write down the profit equation in terms of L,K, and A for this firm and find the first order conditions.

d-describe what you would do to find the profit function for this firm

0 0
Add a comment Improve this question Transcribed image text
Answer #1

come positiv cau then ro duction functim beme メ+B+Y Now if β+Y = 1 then we sal pwdurtim function exhi bitiny nsfant return twe nced to minimise above cot funcfin 1궁 Finding FoC with Yespect t LrfL 44 02 てheJe an G 2 fof ard er 97Proht function When output hao sening pna。P 6、nding ful with rej pect to L/K 4/ A 8下 213 3 ?)下 6 み1( 1 8 A nu 1 win find valunew (つ 一ㄍ L ( し57 レ kで NIL ︵ 324 う TUL し冷 116 V 3 Sy L A2 43 573 36x324 34/1s A 3-9/6 2 3 33 ,リ 더 63 p 3 1x 3メ324n3 variabl cas iF l , t element of Hess, an matriメ< anィ HL such a, | u ル a n Determinant Fo C then value of vari aiJes For

Add a comment
Know the answer?
Add Answer to:
Suppose q=L1/3K1/6A1/18. You get to choose the L,K, and A and the cost of these inputs are w, r, and m, respectively. a-...
Your Answer:

Post as a guest

Your Name:

What's your source?

Earn Coins

Coins can be redeemed for fabulous gifts.

Not the answer you're looking for? Ask your own homework help question. Our experts will answer your question WITHIN MINUTES for Free.
Similar Homework Help Questions
  • Suppose q=L1/3K1/6A1/18. You get to choose the L,K, and A and the cost of these inputs...

    Suppose q=L1/3K1/6A1/18. You get to choose the L,K, and A and the cost of these inputs are w, r, and m, respectively. a- what type of returns to scale does this production function have? b- write down the lagrange for the cost minimization problem and find the first order conditions. c- Write down the profit equation in terms of L,K, and A for this firm and find the first order conditions. d-describe what you would do to find the profit...

  • Consider a textile manufacturing firm that uses labor and capital inputs and has the production technology...

    Consider a textile manufacturing firm that uses labor and capital inputs and has the production technology given by the equation Q = 8K0.25L 0.5 , where Q is output, K is capital and L is labor. Each unit of capital costs 10 TL while each unit of labor costs 5 TL. a) Does this firm have increasing, decreasing or constant returns to scale? (1) b) Define the cost minimization problem faced by firm. What is the objective function, what is...

  • 2. Suppose the production function of a firm is given by q=L1/4K2/4. The prices of labor...

    2. Suppose the production function of a firm is given by q=L1/4K2/4. The prices of labor and capital are given by w = $9 and r= $18, respectively. a) Write down the firm cost minimization formally. b) What returns to scale does the production function exhibit? Explain. c) What is the Marginal Rate of Technical Substitution (MRTS) between capital and labor? d) What is the optimal capital to labor ratio? Show your work.

  • Suppose the production function of a firm is given by q = L1/4K1/4. The prices of...

    Suppose the production function of a firm is given by q = L1/4K1/4. The prices of labor and capital are given by w = $10 and r = $20, respectively. a) Write down the firm's cost minimization problem. b) What returns to scale does the production function exhibit? Explain c) What is the Marginal Rate of Technical Substitution (MRTS) between capital and labor? d) What is the optimal capital to labor ratio? Show your work. e) Derive the long run...

  • NEED ALL ANSWERS PLEASE Problem 3 [24 marks] A competitive firm uses two inputs, capital (k) and labour (), to produce...

    NEED ALL ANSWERS PLEASE Problem 3 [24 marks] A competitive firm uses two inputs, capital (k) and labour (), to produce one output, (y). The price of capital, W, is S1 per unit and the price of labor, wi, is SI per unit. The firm operates in competitive markets for outputs and inputs, so takes the prices as given. The production function is f(k,l) 3k025/025. The maximum amount of output produced for a givern amount of inputs is y(k, l)...

  • A firm has the production function F(L, K) = L1/2 + K1/2. The price of labor...

    A firm has the production function F(L, K) = L1/2 + K1/2. The price of labor is $30 and the price of capital is $10. The firm has a production goal of 100 units of output. a) Carefully write out this firm’s cost minimization problem, using the particulars of this problem. b) Give two equations—particular to this problem—that the solution satisfies. c) Solve for the firm’s optimal input bundle. d) Determine the firm’s cost of producing 100 units of output....

  • The production of Florida strawberries uses two inputs: labor (L) and capital (K). The following production...

    The production of Florida strawberries uses two inputs: labor (L) and capital (K). The following production function describes how these inputs are combined to produce bushels of oranges. f(L,K) = 5(1/2 + 3K1/2 1) Determine what kind of returns to scale this production function exhibits (HINT: labor is the "x" variable - the one that goes on the horizontal axis). 2) What is the formula for that kind of returns to scale? (HINT: use f(L,K)) 3) What is the general...

  • Now suppose that q = L"K". Starting by multiplying the inputs in production by D >...

    Now suppose that q = L"K". Starting by multiplying the inputs in production by D > 1, derive conditions on a and b that determine whether the production function exhibits decreasing, constant and increasing returns to scale.

  • Assume a firm' production function is Q = 3K +L • In this case, inputs (K...

    Assume a firm' production function is Q = 3K +L • In this case, inputs (K and L) are perfect substitutes. Can you give a real example where this production function works? Assume price of capital is r = 5, and price of labor is w = 1 How many units of capital and labor is need to produce Q=60 in cheapest way? O Show your logic using cost minimization condition, and Analyze it graphically

  • 1. Consider the production function y = f(L,K) for a firm in a competitive market setting. The price of the outpu...

    1. Consider the production function y = f(L,K) for a firm in a competitive market setting. The price of the output good is p > 0. The prices of the inputs Labour and Capital are w> 0 and r>0 respectively. The firm chooses L and K in order to maximize profits, (L.K). (a) How does the short-run production function differ from the long-run production function? (b) Write out the profit function for the firm, (L,K). (c) Derive the first order...

ADVERTISEMENT
Free Homework Help App
Download From Google Play
Scan Your Homework
to Get Instant Free Answers
Need Online Homework Help?
Ask a Question
Get Answers For Free
Most questions answered within 3 hours.
ADVERTISEMENT
ADVERTISEMENT
ADVERTISEMENT